Difference between revisions of "2022 AMC 10B Problems/Problem 7"
Songmath20 (talk | contribs) (→Solution 4) |
MRENTHUSIASM (talk | contribs) |
||
(11 intermediate revisions by 8 users not shown) | |||
Line 28: | Line 28: | ||
Multiply by <math>2</math> to account for negative <math>k</math> values (since <math>k</math> is being squared), and our answer is <math>\boxed{\textbf{(B) }8}</math>. | Multiply by <math>2</math> to account for negative <math>k</math> values (since <math>k</math> is being squared), and our answer is <math>\boxed{\textbf{(B) }8}</math>. | ||
− | ==Solution 4 | + | ==Solution 4== |
− | Since <math>36 = | + | Since <math>36 = 2^2\cdot3^2</math>, that means there are <math>(2+1)(2+1) = 9</math> possible factors of <math>36</math>. Since <math>6 \cdot 6</math> violates the distinct root condition, subtract <math>1</math> from <math>9</math> to get <math>8</math>. Each sum is counted twice, and we count of those twice for negatives. This cancels out, so we get <math>\boxed{\textbf{(B) }8}</math>. |
− | ==Video Solution | + | ~songmath20 Edited 5.1.2023 |
+ | |||
+ | ==Video Solution (⚡️Lightning Fast⚡️)== | ||
https://youtu.be/WX871JJbdY4 | https://youtu.be/WX871JJbdY4 | ||
Line 42: | Line 44: | ||
==Video Solution by Interstigation== | ==Video Solution by Interstigation== | ||
https://youtu.be/_KNR0JV5rdI?t=679 | https://youtu.be/_KNR0JV5rdI?t=679 | ||
+ | |||
+ | ==Video Solution by Math4All999== | ||
+ | https://youtube.com/watch?v=cnUq_Op3YzY&feature=shared | ||
+ | |||
+ | ==Video Solution by Gavin Does Math== | ||
+ | https://youtu.be/1qO3eejxuPo | ||
== See Also == | == See Also == |
Latest revision as of 00:09, 1 November 2023
- The following problem is from both the 2022 AMC 10B #7 and 2022 AMC 12B #4, so both problems redirect to this page.
Contents
Problem
For how many values of the constant will the polynomial have two distinct integer roots?
Solution 1
Let and be the roots of By Vieta's Formulas, we have and
It follows that and must be distinct factors of The possibilities of are Each unordered pair gives a unique value of Therefore, there are values of namely
~stevens0209 ~MRENTHUSIASM ~
Solution 2
Note that must be an integer. Using the quadratic formula, Since divides evenly, and have the same parity, so is an integer if and only if is a perfect square.
Let Then, Since is an integer and is even, and must both be even. Assuming that is positive, we get possible values of , namely , which will give distinct positive values of , but gives and , giving identical integer roots. Therefore, there are distinct positive values of Multiplying that by to take the negative values into account, we get values of .
~pianoboy
Solution 3 (Pythagorean Triples)
Proceed similar to Solution 2 and deduce that the discriminant of must be a perfect square greater than to satisfy all given conditions. Seeing something like might remind us of a right triangle, where is the hypotenuse, and is a leg. There are four ways we could have this: a -- triangle, a -- triangle, a -- triangle, and a -- triangle.
Multiply by to account for negative values (since is being squared), and our answer is .
Solution 4
Since , that means there are possible factors of . Since violates the distinct root condition, subtract from to get . Each sum is counted twice, and we count of those twice for negatives. This cancels out, so we get .
~songmath20 Edited 5.1.2023
Video Solution (⚡️Lightning Fast⚡️)
~Education, the Study of Everything
Video Solution(1-16)
~~Hayabusa1
Video Solution by Interstigation
https://youtu.be/_KNR0JV5rdI?t=679
Video Solution by Math4All999
https://youtube.com/watch?v=cnUq_Op3YzY&feature=shared
Video Solution by Gavin Does Math
See Also
2022 AMC 10B (Problems • Answer Key • Resources) | ||
Preceded by Problem 6 |
Followed by Problem 8 | |
1 • 2 • 3 • 4 • 5 • 6 • 7 • 8 • 9 • 10 • 11 • 12 • 13 • 14 • 15 • 16 • 17 • 18 • 19 • 20 • 21 • 22 • 23 • 24 • 25 | ||
All AMC 10 Problems and Solutions |
2022 AMC 12B (Problems • Answer Key • Resources) | |
Preceded by Problem 3 |
Followed by Problem 5 |
1 • 2 • 3 • 4 • 5 • 6 • 7 • 8 • 9 • 10 • 11 • 12 • 13 • 14 • 15 • 16 • 17 • 18 • 19 • 20 • 21 • 22 • 23 • 24 • 25 | |
All AMC 12 Problems and Solutions |
The problems on this page are copyrighted by the Mathematical Association of America's American Mathematics Competitions.